Show that a mapping is continuous

  • Thread starter Thread starter DeadOriginal
  • Start date Start date
  • Tags Tags
    Continuous Mapping
Click For Summary
The discussion revolves around proving the continuity of a mapping that transforms points in E^{n+1}-0 to a new format. Participants reference three definitions of continuity, emphasizing the importance of demonstrating that the inverse image of an open set is also open. One user suggests using the first definition to show that a sequence converges to a limit under the transformation. Clarification is sought on how to identify limit points and the necessity of the assumption that a sequence converges. The conversation highlights the need for a structured approach to proving continuity through limit points and open sets.
DeadOriginal
Messages
274
Reaction score
2

Homework Statement


Show that the mapping f carrying each point (x_{1},x_{2},...,x_{n+1}) of E^{n+1}-0 onto the point (\frac{x_{1}}{|x|^{2}},...,\frac{x_{n+1}}{|x|^{2}}) is continuous.

2. Continuity theorems I am given.
A transformation f:S->T is continuous provided that if p is a limit point of a subset X of S then f(p) is a limit point or a point of f(X).

Let f:S->T be a transformation of space S into space T. A necessary and sufficient condition that f be continuous is that if O is an open subset of T, then its inverse image f^{-1}(O) is open in S.

A necessary an sufficient condition that the transformation f:S->T be continuous is that if x is a point of S, and V is an open subset of T containing f(x) then there is an open set U in S containing x and such that f(U) lies in V.

The Attempt at a Solution


I was thinking to prove this I would have to find an open set in the range of this function and show that its inverse image is also open in the domain but I am not sure how I would go about doing that.

Any help would be appreciated.
 
Last edited:
Physics news on Phys.org
What is the definition of continuity you work with?
 
I have three. They are

A transformation f:S->T is continuous provided that if p is a limit point of a subset X of S then f(p) is a limit point or a point of f(X).

Let f:S->T be a transformation of space S into space T. A necessary and sufficient condition that f be continuous is that if O is an open subset of T, then its inverse image f^(-1)(O) is open in S.

A necessary an sufficient condition that the transformation f:S->T be continuous is that if x is a point of S, and V is an open subset of T containing f(x) then there is an open set U in S containing x and such that f(U) lies in V.

I updated the original post to reflect this.
 
I would use the first one. Given a sequence ## \mathbf{x}^{(n)} \rightarrow \mathbf{p} ##, prove that ## \frac {\mathbf{x}^{(n)}} {(x^{(n)})^2} \rightarrow \frac {\mathbf{p}} {p^2} ##.
 
voko said:
I would use the first one. Given a sequence ## \mathbf{x}^{(n)} \rightarrow \mathbf{p} ##, prove that ## \frac {\mathbf{x}^{(n)}} {(x^{(n)})^2} \rightarrow \frac {\mathbf{p}} {p^2} ##.

I don't really understand. Can you elaborate a little bit? I don't know how I would show that a point x=(x_{1},x_{2},...,x_{n+1}) is a limit point.
 
You don't have to prove ## \mathbf{x}^{(n)} \rightarrow \mathbf{p} ##, this is an assumption. Given that assumption, prove ## \frac {\mathbf{x}^{(n)}} {(x^{(n)})^2} \rightarrow \frac {\mathbf{p}} {p^2} ##.
 
Question: A clock's minute hand has length 4 and its hour hand has length 3. What is the distance between the tips at the moment when it is increasing most rapidly?(Putnam Exam Question) Answer: Making assumption that both the hands moves at constant angular velocities, the answer is ## \sqrt{7} .## But don't you think this assumption is somewhat doubtful and wrong?

Similar threads

Replies
6
Views
2K
Replies
2
Views
2K
  • · Replies 12 ·
Replies
12
Views
2K
  • · Replies 5 ·
Replies
5
Views
2K
Replies
23
Views
3K
  • · Replies 105 ·
4
Replies
105
Views
6K
  • · Replies 5 ·
Replies
5
Views
1K
Replies
4
Views
2K
  • · Replies 1 ·
Replies
1
Views
2K
  • · Replies 4 ·
Replies
4
Views
2K